Menu Close

Category: Limits

lim-x-0-x-2-1-1-5-x-1-1-3-x-1-1-3-x-1-

Question Number 102036 by Study last updated on 06/Jul/20 $${li}\underset{{x}\rightarrow\mathrm{0}} {{m}}\frac{\sqrt[{\mathrm{5}}]{{x}^{\mathrm{2}} −\mathrm{1}}\:+\sqrt[{\mathrm{3}}]{{x}+\mathrm{1}}}{\:\sqrt[{\mathrm{3}}]{{x}−\mathrm{1}}\:+\sqrt{{x}+\mathrm{1}}}=? \\ $$ Answered by john santu last updated on 06/Jul/20 $${L}'{Hopital}\: \\ $$$$\underset{{x}\rightarrow\mathrm{0}}…

lim-x-2-2x-4-1-3-2-x-2-x-2-

Question Number 102034 by Study last updated on 06/Jul/20 $${li}\underset{{x}\rightarrow\mathrm{2}} {{m}}\frac{\sqrt[{\mathrm{3}}]{\mathrm{2}{x}+\mathrm{4}}\:−\mathrm{2}}{{x}^{\mathrm{2}} −{x}−\mathrm{2}}=? \\ $$ Answered by Dwaipayan Shikari last updated on 06/Jul/20 $$\underset{{x}\rightarrow\mathrm{2}} {\mathrm{lim}}\frac{\sqrt[{\mathrm{3}}]{\mathrm{2}{x}+\mathrm{4}}−\mathrm{2}}{\mathrm{2}{x}+\mathrm{4}−\mathrm{8}}.\frac{\mathrm{2}{x}−\mathrm{4}}{\left({x}−\mathrm{2}\right)\left({x}+\mathrm{1}\right)}=\frac{\mathrm{1}}{\mathrm{3}}\mathrm{8}^{\frac{−\mathrm{2}}{\mathrm{3}}} .\frac{\mathrm{2}}{\mathrm{3}}=\frac{\mathrm{1}}{\mathrm{18}}…

lim-x-0-tan-pi-4-x-1-x-

Question Number 102020 by Dwaipayan Shikari last updated on 06/Jul/20 $$\underset{{x}\rightarrow\mathrm{0}} {\mathrm{lim}}\left({tan}\left(\frac{\pi}{\mathrm{4}}−{x}\right)\right)^{\frac{\mathrm{1}}{{x}}} \\ $$ Answered by john santu last updated on 06/Jul/20 $$\underset{{x}\rightarrow\mathrm{0}} {\mathrm{lim}}\left(\mathrm{1}+\left(\mathrm{tan}\:\left(\frac{\pi}{\mathrm{4}}−{x}\right)−\mathrm{1}\right)\right)^{\frac{\mathrm{1}}{{x}}} \\…

lim-x-0-tanx-x-1-x-2-

Question Number 101981 by Rohit@Thakur last updated on 05/Jul/20 $${li}\underset{{x}\rightarrow\mathrm{0}} {{m}}\left(\frac{{tanx}}{{x}}\right)^{\frac{\mathrm{1}}{{x}^{\mathrm{2}} }} \\ $$ Answered by Dwaipayan Shikari last updated on 06/Jul/20 $$\underset{{x}\rightarrow\mathrm{0}} {\mathrm{lim}}\left(\frac{{tanx}}{{x}}\right)^{\frac{\mathrm{1}}{{x}^{\mathrm{2}} }}…

Given-2-functions-f-and-g-n-times-derivable-within-the-open-interval-R-and-verify-the-property-f-x-0-f-k-x-0-0-g-x-0-g-k-x-0-0-k-1-2-n-1-Show-that-lim-x-x-0-f-x-

Question Number 101977 by Ar Brandon last updated on 05/Jul/20 $$\mathrm{Given}\:\mathrm{2}\:\mathrm{functions},\:\mathrm{f}\:\mathrm{and}\:\mathrm{g},\:\mathrm{n}-\mathrm{times}\:\mathrm{derivable}\:\mathrm{within} \\ $$$$\mathrm{the}\:\mathrm{open}\:\mathrm{interval},\:\mathbb{R}\:\mathrm{and}\:\mathrm{verify}\:\mathrm{the}\:\mathrm{property} \\ $$$$\mathrm{f}\left(\mathrm{x}_{\mathrm{0}} \right)=\mathrm{f}^{\left(\mathrm{k}\right)} \left(\mathrm{x}_{\mathrm{0}} \right)=\mathrm{0}\:,\:\mathrm{g}\left(\mathrm{x}_{\mathrm{0}} \right)=\mathrm{g}^{\left(\mathrm{k}\right)} \left(\mathrm{x}_{\mathrm{0}} \right)=\mathrm{0}\:,\:\forall\mathrm{k}\in\left\{\mathrm{1},\mathrm{2},…,\mathrm{n}−\mathrm{1}\right\} \\ $$$$\mathrm{Show}\:\mathrm{that}\:\underset{\mathrm{x}\rightarrow\mathrm{x}_{\mathrm{0}} } {\mathrm{lim}}\frac{\mathrm{f}\left(\mathrm{x}\right)}{\mathrm{g}\left(\mathrm{x}\right)}=\frac{\mathrm{f}^{\left(\mathrm{n}\right)}…

Question-167421

Question Number 167421 by infinityaction last updated on 16/Mar/22 Answered by mindispower last updated on 16/Mar/22 $${cot}^{\mathrm{2}} \left(\frac{{k}\pi}{\mathrm{2}{n}+\mathrm{1}}\right)={cot}^{\mathrm{2}} \left({s}\pi\right)=\left({i}\frac{{e}^{{is}} +{e}^{−{is}} }{{e}^{{is}} −{e}^{−{is}} }\right)^{\mathrm{2}} = \\…

lim-n-n-1-n-1-n-n-JS-

Question Number 101822 by john santu last updated on 04/Jul/20 $$\underset{{n}\rightarrow\infty} {\mathrm{lim}}\frac{\phi^{{n}+\mathrm{1}} −\left(−\phi\right)^{−{n}−\mathrm{1}} }{\phi^{{n}} −\left(−\phi\right)^{−{n}} }\:=\: \\ $$$$\left({JS}\:\circledast\right) \\ $$ Answered by bobhans last updated…

Question-167350

Question Number 167350 by infinityaction last updated on 13/Mar/22 Answered by Jamshidbek last updated on 13/Mar/22 $$\mathrm{Telegram}:@\mathrm{math\_undergraduate} \\ $$$$\mathrm{Problem}\:\mathrm{15}. \\ $$$$\mathrm{This}\:\mathrm{has}\:\mathrm{problem}\:\mathrm{solution}\:\mathrm{in}\:\mathrm{telegram}\:\mathrm{channel} \\ $$ Commented by…